Your-Doctor
Multiple Choice Questions (MCQ)



Free Palestine
Quiz Categories Click to expand

Category: Q&A Medicine--->Cardiology
Page: 9

Question 41# Print Question

A 62-year-old Caucasian man presents to the clinic for his annual physical examination. The patient has a history of hypertension, which is adequately controlled with medications and lifestyle changes. The patient reluctantly admits that he still smokes a half pack of cigarettes daily. On examination, his blood pressure is 132/88 mmHg. During the physical examination, a pulsatile abdominal mass is noted. The patient denies any symptoms from the mass. An ultrasound is ordered and reveals that the abdominal aortic diameter is 4.5 cm.

What is the next step in management?

A. Observation
B. Rapid reduction in blood pressure
C. Immediate surgery
D. Surgery within the next 48 hours


Question 42# Print Question

A 68-year-old woman presents to the Emergency Department with substernal chest pain radiating to her jaw. Her medical history is significant for hypertension and hyperlipidemia. Troponins are positive, and an ECG confirms STEMI. She undergoes percutaneous coronary intervention. The rest of her hospitalization is unremarkable, and she is eventually discharged. She returns to your office 1 month later complaining of a persistent dry cough without any other symptoms. Physical examination is unremarkable.

Which of the following is the most appropriate next step in management?

A. Check troponin levels and refer for stress test
B. Stop the patient’s lisinopril and start losartan
C. Order a chest x-ray
D. Stop the patient’s aspirin and start bronchodilator therapy
E. Reassurance


Question 43# Print Question

A 63-year-old woman presents to the hospital with shortness of breath and palpitations that started this morning. She has a history of coronary artery disease and takes appropriate medications. Her ECG is shown below (Figure below).

Which of the following is the most likely diagnosis?

A. Atrial fibrillation
B. Atrial flutter
C. Multifocal atrial tachycardia
D. Atrioventricular nodal reentrant tachycardia (AVNRT)


Question 44# Print Question

A 59-year-old man with a history of nonischemic cardiomyopathy presents to the hospital with leg swelling and extreme shortness of breath. He reports that several weeks ago, he had trouble obtaining his medications due to losing his medical insurance. He normally takes losartan, carvedilol, nifedipine, aspirin, furosemide, atorvastatin, insulin, and omeprazole. His vitals show a blood pressure of 104/64 mmHg, a heart rate of 72 beats per minute, and a respiratory rate of 28 breaths per minute. He has an S3 on examination with bilateral rales at his lung bases and 2+ pitting edema of his lower extremities up to his sacrum. An ECG shows no acute ST elevations or depressions. A chest x-ray shows bilateral fluffy opacities, and an echocardiogram shows an estimated ejection fraction of 20%.

Which of the following medications should not be resumed at this time?

A. Furosemide
B. Insulin
C. Losartan
D. Atorvastatin
E. Carvedilol


Question 45# Print Question

A 55-year-old man presents to the hospital with progressive dyspnea over the course of the last few weeks. He denies fever, cough, or chest pain. He has no significant medical or family history, but endorses drinking two 750-mL bottles of wine daily. He is afebrile with a blood pressure of 139/82 mmHg, heart rate of 98 beats per minute, and respiratory rate of 24 breaths per minute. The point of maximal impulse is displaced laterally, a 2/6 holosystolic murmur is heard at the apex, and an S3 is auscultated. There are crackles at the lung bases with pitting edema around the ankles.

Which of the following is the most important intervention for this patient?

A. Low sodium diet
B. Cardiac glycoside
C. β-blocker
D. Alcohol cessation




Category: Q&A Medicine--->Cardiology
Page: 9 of 12